宏觀經濟學

無限視界橫向條件

  • May 6, 2016

這個問題是內生增長的擴展:使用 CRRA 實用程序的平衡增長路徑

然而,這個問題詢問了該問題中使用的特定概念,我認為有一個專門討論這個概念的問題會很有幫助。

我們將在這個問題中使用模型:

$ \textbf{Model:} $

$$ K_t=\frac{1}{n}\sum_{t=1}^nk_t $$ 在這個模型中, $ k_t $ 由代理人選擇,並且 $ K_t=\bar{k}_t $ (所有的平均值 $ k_t $ ). 現在,代理人想要動態地最大化效用(在某些約束下)並且他們有 CRRA(恆定的相對風險厭惡)效用,所以最大化看起來像:

$$ \sum_{t=0}^\infty\beta^t\bigg(\frac{c_t^{1-\gamma}}{1-\gamma}\bigg) $$ $$ s.t.;Y_t=k_t^\alpha(E_tL)^{1-\alpha} $$ $$ c_t+i_t=Y_t $$ $$ k_{t+1}=(1-\delta)k_t+i_t $$ $$ c_t,i_t\geq0 $$ $ E_tL $ 是有效勞動,其餘變數是典型的(如果需要,我可以給出它們的定義)。

模型的最後一個補充是有兩個平衡約束:

$$ E_t=\frac{K_t}{L} $$ $$ k_t=K_t $$

在答案部分,我被指示使用橫向條件和 inada 條件,以表明平衡的增長路徑是最佳的。以下是我對橫向條件的推導:

首先,我們必須推導出有限視界橫向條件。我們通過解決以下問題來做到這一點:

$$ \underset{{k_t}{t=0}^{T+1}}{max};\sum{t=0}^{T}\beta^tU(\frac{}{}) $$ $$ s.t.;k_{T+1}\geq0 $$ 對於我們的模型,這意味著: $$ \underset{{k_t}{t=0}^{T+1}}{max};\sum{t=0}^{T}\beta^t\bigg(\frac{((1+1-\delta)k_t-k_{t+1})^{1-\gamma}}{1-\gamma}\bigg) $$ $$ s.t.;k_{T+1}\geq0 $$ 我們的拉格朗日是:

$$ \ell(\frac{}{})=\sum_{t=0}^{T}\beta^t\bigg(\frac{((1+1-\delta)k_t-k_{t+1})^{1-\gamma}}{1-\gamma}\bigg)+\lambda k_{T+1} $$ 為了得到橫向條件,我們對 $ k_{T+1} $ 並使用約束。

火:

$$ \frac{\beta^T}{((1+(1-\delta))k_T-k_{T+1})^\gamma}=\lambda \qquad (1) $$ $$ \lambda k_{T+1}=0 \qquad (2) $$ $$ \lambda , k_{T+1}\geq 0 \qquad (3) $$ 解決這個系統,我們得到 $ \textbf{the finite horizon transversality condition is:} $ $$ \bigg(\frac{\beta^T}{((1+(1-\delta))k_T-k_{T+1})^\gamma}\bigg)k_{T+1}=0 $$ 現在,為了得到無限地平線橫向條件,我們再次最大化貼現效用函式,但我們將有限地平線橫向條件的極限設置為 $ T\rightarrow \infty $ 等於 0 作為約束。 這意味著我們必須解決:

$$ \underset{{k_t}{t=0}^\infty}{max};\sum{t=0}^\infty\beta^t\bigg(\frac{((1+(1-\delta))k_t-k_{t+1})^{1-\gamma}}{1-\gamma}\bigg) $$ $$ s.t.; \underset{T\rightarrow\infty}{lim};\bigg[\bigg(\frac{\beta^T}{((1+(1-\delta))k_T-k_{T+1})^\gamma}\bigg)k_{T+1}\bigg]=0 $$ 這就是我的問題所在。我們如何解決這個問題?我們可以使用拉格朗日嗎?如果是這樣,我們如何在存在限制的情況下採用一階條件?任何幫助將不勝感激!

您的困惑來自於您將 Transversality 條件視為約束,而它是最優性的條件。所以你問題末尾的表述是錯誤的。你要做的是像往常一樣求解你的模型,然後檢查解決方案(這裡是平衡增長路徑)是否滿足橫向條件。

橫向條件 (TVC) 是

$$ \lim_{t \rightarrow \infty} \beta^t u’(c_t)k_{t+1} =0 $$ 在你的模型中, $ u’(c_t) = c_t^{-\gamma} $ 和 $ c_t = (2-\delta)k_t - k_{t+1} $

所以TVC確實變成了

$$ \lim_{t \rightarrow \infty} \frac {\beta^tk_{t+1}}{[(2-\delta)k_t - k_{t+1}]^{\gamma}} =0 $$ 我們想要的是檢查(唯一的)平衡增長路徑是否滿足橫向條件。在我們擁有的平衡增長道路上 $ k_{t+1} = (1+g)k_t $ 在哪裡 $ g $ 是由模型的外生參數和優化條件決定的常數。因此,當處於平衡增長路徑上時,TVC 變為

$$ \lim_{t \rightarrow \infty} \frac {\beta^t(1+g)k_t}{[(2-\delta)k_t - (1+g)k_t]^{\gamma}} = \lim_{t \rightarrow \infty}\frac {(1+g)}{(1-\delta-g)^{\gamma}}\beta^t k_t^{1-\gamma}=0 $$ 常量並不重要,所以我們想要

$$ \lim_{t \rightarrow \infty}\beta^t k_t^{1-\gamma}=0 $$ 持有。請注意,如果 $ \gamma \geq 1 $ (這是文獻中的共識)條件確實成立,因為資本進入分母,我們就完成了。

對於案件 $ \gamma <1 $ ,我們希望表達式只保持在極限,即最終。“最終”意味著,如果 $ k^_s $ 是當時的資本水平 $ s<t $ 當經濟到達平衡增長路徑時,我們將有 $ k_t = k_s^\cdot (1+g)^{t-s} $ . 那麼TVC就變成了

$$ \lim_{t \rightarrow \infty}\beta^t \big[k_s^*\cdot (1+g)^{t-s}\big]^{1-\gamma}=0 $$ 再次,擺脫不依賴的東西 $ t $ , 要得到

$$ \lim_{t \rightarrow \infty}\beta^t (1+g)^{(1-\gamma)t}=0 \implies \lim_{t \rightarrow \infty}\big[\beta (1+g)^{(1-\gamma)}\big]^t=0 $$ 因此,當且僅當 TVC 將成立

$$ \beta (1+g)^{(1-\gamma)} <1 $$ 在另一個問題中,OP已獲得

$$ 1+g = [\beta(\alpha+1-\delta)]^{\frac{1}{\gamma}} $$ 代替,我們想要

$$ \beta \Big([\beta(\alpha+1-\delta)]^{1/\gamma}\Big)^{(1-\gamma)} <1 $$ 這可以稍微簡化一下,它是對平衡增長路徑滿足橫向條件(當 $ \gamma <1 $ )。對於其他參數的基線值,可以看出該條件不太可能得到滿足。

引用自:https://economics.stackexchange.com/questions/11865